Vector Analysis Homework: Finding the Unit Normal Vector at P

Click For Summary
To find the unit normal vector at point P on the surface S defined by w=f(x,y), two non-collinear tangent vectors (u and v) are utilized. The tangent vector u is derived from a plane parallel to the xz-plane that intersects the surface, tracing a curve C at point P. The z-component of vector u is approximated as (∂f/∂x)ux, which is based on a linear approximation of the surface near P. This approximation is valid for small displacements and can be expressed using a truncated Taylor expansion. Understanding this concept is essential for solving vector analysis problems involving surfaces.
manimaran1605
Messages
60
Reaction score
0

Homework Statement



This picture is taken from Div, curl, grad and all that by schey, while finding the unit normal vector of the surface S (defined as w=f(x,y)) at a point P, to find the normal vector he considered a two tangent non-collinear vectors (u and v) at a point P, to find u he considered a plane passing through point P parallel to xz plane, the plane which intersects the surface S traces a curve C which contains the point P, he drawn a tangent to the curve at the point p, let the x component be ux, My question is how the z-component of u is (∂f/∂x)ux ?

Homework Equations


No equations found


The Attempt at a Solution


I have an idea that z-component of u is some approximation, but i havn't learn multivariable calculas a lot, so please enlighten me. Thank you
 

Attachments

  • Untitled-1.jpg
    Untitled-1.jpg
    5.5 KB · Views: 426
Physics news on Phys.org
You are correct. In a small displacement from P, the linear approximation is a good one and you can write ##z(x= P + u_x) \approx z(x=P) + \frac{\partial z}{\partial x} u_x## (The Taylor expansion truncated, ignoring higher order terms). For an infintesimal displacement, this approximation becomes exact.
 
Last edited:
  • Like
Likes 1 person
Question: A clock's minute hand has length 4 and its hour hand has length 3. What is the distance between the tips at the moment when it is increasing most rapidly?(Putnam Exam Question) Answer: Making assumption that both the hands moves at constant angular velocities, the answer is ## \sqrt{7} .## But don't you think this assumption is somewhat doubtful and wrong?

Similar threads

  • · Replies 1 ·
Replies
1
Views
2K
Replies
1
Views
1K
  • · Replies 5 ·
Replies
5
Views
2K
  • · Replies 8 ·
Replies
8
Views
2K
  • · Replies 4 ·
Replies
4
Views
2K
  • · Replies 2 ·
Replies
2
Views
3K
  • · Replies 8 ·
Replies
8
Views
2K
  • · Replies 3 ·
Replies
3
Views
2K
Replies
4
Views
2K
  • · Replies 85 ·
3
Replies
85
Views
10K